exercise

69

Question 1.146

Calculate the dot product of \({\bf x}=(1,-1,0,2)\in {\mathbb R}^4\) and \({\bf y}=(1,2,3,4)\in{\mathbb R}^4\).

Question 1.147

In \({\mathbb R}^n\) show that

  • (a) \(2 \|{\bf x} \|^2 + 2 \|{\bf y}\|^2 = \| {\bf x} + {\bf y}\|^2 + \|{\bf x} - {\bf y} \|^2\) (This is known as the parallelogram law.)
  • (b) \(\|{\bf x} - {\bf y} \|\| {\bf x} + {\bf y}\|\le \|{\bf x} \|^2 + \|{\bf y} \|^2\)
  • (c) \(4 \langle {\bf x}, {\bf y}\rangle = \|{\bf x} + {\bf y} \|^2 - \|{\bf x} - {\bf y}\|^2\) (This is called the polarization identity.)

Interpret these results geometrically in terms of the parallelogram formed by x and y.

Verify the Cauchy–Schwarz inequality and the triangle inequality for the vectors in Exercises 3 to 6.

Question 1.148

\({\bf x} = (2,0,-1), {\bf y} = (4,0,-2)\)

Question 1.149

\({\bf x} = (1,0,2,6), {\bf y} = (3,8,4,1)\)

Question 1.150

\({\bf x} = (1,-1,1, -1,1), {\bf y} =(3,0,0,0, 2)\)

Question 1.151

\({\bf x}=(1,0,0,1),{\bf y}=(-1,0,0,1)\)

Question 1.152

Let \(\textbf{v}, \textbf{w} \in \mathbb{R}^n\). If \( \| \textbf{v} \| = \| \textbf{w} \|\), show that \(\textbf{v} + \textbf{w}\) and \(\textbf{v} - \textbf{w}\) are orthogonal.

Question 1.153

Suppose \(T\) is a triangle formed by placing three points on a circle, two of which lie on the circle’s diameter. Use the previous problem to show \(T\) is a right triangle.

Question 1.154

Compute \(AB, \hbox{det } A, \hbox{det } B, \hbox{det } (A B)\), and \(\hbox{det } (A+B)\) for \[ A = \Bigg[\begin{array}{@{}c@{\quad}r@{\quad}c@{}} 1 & -1 & 0 \\ 0 & 3 & 2 \\ 3 & 1 & 1 \end{array} \Bigg] \qquad \hbox{and } \qquad B = \Bigg[\begin{array}{@{}r@{\quad}c@{\quad}r@{}} -2 & 0 & 2 \\ -1 & 1 & -1 \\ 1 & 4 & 3 \end{array} \Bigg]. \]

Question 1.155

Compute \(AB, \hbox{det } A, \hbox{det } B, \hbox{det } (AB)\), and \(\hbox{det } (A+B)\) for \[ A = \Bigg[\begin{array}{@{}c@{\quad}c@{\quad}r@{}} 3 & 0 & 1\\ 1 & 2 & -1 \\ 1 & 0 & 1 \end{array} \Bigg] \qquad \hbox{and } \qquad B = \Bigg[\begin{array}{@{}c@{\quad}c@{\quad}r@{}} 1 & 0 & -1 \\ 2 & 0 & 1 \\ 0 & 1 & 0 \end{array} \Bigg]. \]

Question 1.156

Determine which of the following matrices are invertible: \[ A=\left[ \begin{array}{ccc} 1 & 2 & 3 \\ 0 & 1 & 1 \\ 0 & 3 & 3 \\ \end{array} \right] \quad B=\left[ \begin{array}{ccc} 0 & 0 & 3 \\ -1 & 1 & 19 \\ 2 & 3 & \pi \\ \end{array} \right] \quad C=\left[ \begin{array}{cc} 1 & 1 \\ 1 & 1 \\ \end{array} \right] \]

Question 1.157

For matrix \(A\) in the previous problem, find a nonzero \(\textbf{x} \in \mathbb{R}^3\) such that \(A\textbf{x}=\textbf{0}\).

Question 1.158

Use induction on \(k\) to prove that if \({\bf x}_1, \ldots , {\bf x}_k \in {\mathbb R}^n\), then \[ \|{\bf x}_1 + \cdots + {\bf x}_k\| \le \|{\bf x}_1 \| + \cdots + \|{\bf x}_k\|. \]

Question 1.159

Using algebra, prove the identity of Lagrange: For real numbers \(x_1, \ldots , x_n\) and \(y_1, \ldots , y_n.\) \[ \bigg( \sum_{i=1}^n x_i y_i \bigg)^2\! = \bigg( \sum_{i=1}^n x_i^2 \bigg) \bigg( \sum_{i=1}^n y_i^2 \bigg) {-} \sum_{i < j} (x_i y_j - x_j y_i)^2. \]

Use this to give another proof of the Cauchy–Schwarz inequality in \({\mathbb R}^n\),

Question 1.160

Prove that if \(A\) is an \(n \times n \) matrix, then

  • (a) \(\hbox{det } ( \lambda A) = \lambda^n \hbox{ det } A;\) and
  • (b) if \(B\) is a matrix obtained from \(A\) by multiplying any row or column by a scalar \(\lambda\), then \(\hbox{det } B = \lambda \hbox{ det } A\).

In Exercises 16 to 18, \(A\),\(B\), and C denote \(n \times n\) matrices.

Question 1.161

Is \(\hbox{det } (A+B) = \hbox{det } A + \hbox{det } B\)? Give a proof or counterexample.

Question 1.162

Does \((A+B) (A-B) = A^2 - B^2\)?

Question 1.163

Assuming the law \(\hbox{det } (AB) = ( \hbox{det } A)(\hbox{det } B)\), prove that det \((ABC) = ( \hbox{det } A) ( \hbox{det } B)(\hbox{det } C)\).

Question 1.164

(This exercise assumes a knowledge of integration of continuous functions of one variable.) Note that the proof of the Cauchy–Schwarz inequality (Theorem 4) depends only on the properties of the inner product listed in Theorem 1. Use this observation to establish the following inequality for continuous functions \(f,g \colon [0,1] \to {\mathbb R}\): \[ \bigg| \int_0^1 f (x) g (x)\ \ dx \bigg| \le \sqrt{ \int^1_0 [f (x) ]^2\ \ dx} \ \sqrt{\int_0^1[ g (x)]^2\ \ dx}. \]

Do this by

  • (a) verifying that the space of continuous functions from [0, 1] to \({\mathbb R}\) forms a vector space; that is, we may think of functions \(f,g\) abstractly as “vectors” that can be added to each other and multiplied by scalars.
  • (b) introducing the inner product of functions \[ f \,{\cdot}\, g = \int_0^1 f (x) g (x) \ dx \] and verifying that it satisfies conditions (i) to (iv) of Theorem 3.

Question 1.165

70

Define the transpose \(A^T\) of an \(n \times n\) matrix \(A\) as follows: the \(ij\)th element of \(A^T\) is \(a_{ji}\) where \(a_{ij}\) is the \(ij\)th entry of \(A\). Show that \(A^T\) is characterized by the following property: For all \({\bf x,y}\) in \({\mathbb R}^n\), \[ (A^T {\bf x}) \,{\cdot}\, {\bf y} = {\bf x} \,{\cdot}\, ( A {\bf y}). \]

Question 1.166

Verify that the inverse of \[ \Big[\begin{array} &a & b \\ c & d \end{array} \Big] \qquad \hbox{is} \qquad \frac{1}{ad- bc} \Big[\begin{array}{@{}r@{\quad}r@{}} d & -b \\ -c & a \end{array} \Big]. \]

Question 1.167

Use your answer in Exercise 21 to show that the solution of the system \begin{eqnarray*} && ax+ by = e \\ && cx + dy =f\\[-21.5pt] \end{eqnarray*} is \[ \Big[\begin{array}{c} x \\ y \end{array} \Big] = \frac{1}{ad-bc} \Big[\begin{array}{@{}r@{\quad}r@{}} d & -b \\ -c & a \end{array} \Big] \Big[\begin{array}{c} e \\ f \end{array} \Big]. \]

Question 1.168

Assuming the law \(\hbox{det } (AB) = ( \hbox{det } A) ( \hbox{det } B)\), verify that \((\hbox{det } A) (\hbox{det } A^{-1})=1\) and conclude that if \(A\) has an inverse, then \(\hbox{det } A \ne 0\).

Question 1.169

Find two \(2 \times 2\) matrices \(A\) and \(B\) such that \(AB=0\) but \(BA \neq 0\).